Harmonic Divisions - A Powerful & Rarely Used Tool!

by Potla, Feb 8, 2011, 5:56 PM

Long since I have ever posted anything on my blog, and more importantly, I have not posted anything on geometry before (except for a similarity trivia), and therefore this is my first extensive attempt on geometry.
As the title explains clearly, I want to post some useful results and solve some problems related to Harmonic Divisions, because I have not found a very good book or ebook explaining the concepts clearly. :)
HARMONIC DIVISIONS
Though all of us know what a harmonic division is, if four points $A,B,C,D$ are collinear and if they satisfy
$\frac{AC}{CB}=\frac {AD}{DB},$ then the four points $ACBD$ are said to form a harmonic range.
Now let us move onto some theorems.
Theorem 1.
If $(ACBD)$ is a harmonic range and if $O$ is the midpoint of $AB$ then $OB^2=OC\cdot OD.$
Proof.
After componendo and dividendo on $\frac{AC}{CB}=\frac{AD}{DB};$ we get $\frac{AC+CB}{AC-CB}=\frac{AD+DB}{AD-DB};$ leading to
$\frac{2OB}{2OC}=\frac{2OD}{2OB}\iff OB^2=OC\cdot OD.$
The converse of this theorem is also obviously true.
[asy]
dot((0,0));
dot((-5,0));
dot((+5,0));
dot((3,0));
dot((8,0));
draw((-5,0)--(8,0));
label("$A$", (-5,0), S);
label("$O$", (0,0), S);
label("$C$", (3,0), S);
label("$D$", (8,0), S);
label("$B$", (5,0), S);
[/asy]
Theorem 2.
If $AX, BY, CZ$ are three concurrent cevians of a triangle $\triangle ABC$ with $X, Y, Z$ lying on $BC, CA, AB$ respectively, then $ZY\cap BC=T\implies (BXCT)$ is a harmonic division.
Proof
Using Menelaus we have,
$\frac{|AZ|}{|BZ|}\cdot\frac{|BT|}{|CT|}\cdot \frac{|CY|}{|AY|}=1;$
And using Ceva we obtain
$\frac{|AZ|}{|BZ|}\cdot\frac{|BX|}{|CX|}\cdot\frac{|CY|}{|AY|}=1;$
Which jointly lead to $\frac{|BX|}{|CX|}=\frac{|BT|}{|CT|}\implies (BXCT)=-1;$ ie the division is harmonic.
[asy]
size(10cm, 8cm);
label("$A$", (0,5), N);
label("$B$", (-1,0), S);
label("$C$", (5,0), S);
label("$X$", (3,0), S);
label("$Z$", (-0.3, 3.48), NW);
label("$Y$", (2.35,2.65), NE);
label("$T$", (10.82,0), S);
draw((0,5)--(-1,0));
draw((-1,0)--(10.82,0));
draw((5,0)--(0,5));
draw((0,5)--(3,0));
draw((-1,0)--(2.35,2.65));
draw((5,0)--(-0.3,3.48));
draw((-0.3,3.48)--(10.82,0));
[/asy]
Theorem 3.
If four concurrent lines are such that one transversal cuts them harmonically, then every transversal intersects these four lines to form a harmonic division.
Proof
Assume the lines $OA, OB, OC, OD$ pass through $O$ and cut by a line $d$ in $A,B,C,D$ respectively. Assume that $(ACBD)$ is harmonic, and therefore let another transversal cut these at $A',B',C',D';$ it is sufficient to check that the four points $A', B', C', D'$ form a harmonic division.
Note that using the sine rule we get
$\frac{AC}{CB}=\frac{OA}{OB}\cdot\frac{\sin\angle AOC}{\sin\angle COB};$ and $\frac{AD}{BD}=\frac{OA}{OB}\cdot\frac{\sin\angle AOD}{\sin\angle BOD}.$
Comparing these two and using the fact that $(ACBD)$ is harmonic, we get $\frac{\sin\angle AOC}{\sin\angle COB}=\frac{\sin\angle AOD}{\sin\angle BOD}.$
Now again we can use similar arguments to arrive at $\frac{A'C'}{C'B'}=\frac{A'D'}{B'D'};$ ie $(A'C'B'D')$ is harmonic.
[asy]
import graph; size(300); real lsf = 0.5; pen dp = linewidth(0.5) + fontsize(10); defaultpen(dp); pen ds = black; pen xdxdff = rgb(0.49,0.49,1);
draw((-2,0)--(4,0)); draw((2,0)--(10,0)); draw((0,(+12-6*0)/2)--(14.29,(+12-6*14.29)/2)); draw((0,(+24-6*0)/4)--(14.29,(+24-6*14.29)/4)); draw((0,(+60-6*0)/10)--(14.29,(+60-6*14.29)/10)); draw((-2,(+12+6*-2)/2)--(14.29,(+12+6*14.29)/2)); draw((1.33,(+14.29-5.23*1.33)/0.73)--(14.29,(+14.29-5.23*14.29)/0.73));
dot((-2,0),ds); label("$A$", (-2.17,-0.56),S*lsf); dot((4,0),ds); label("$B$", (3.81,-0.47),S*lsf); dot((2,0),ds); label("$C$", (1.67,-0.53),SW*lsf); dot((10,0),ds); label("$D$", (9.83,-0.5),SE*lsf); dot((0,6),ds); label("$O$", (-0.65,6.3),NE*lsf); dot((1.33,9.99),ds); label("$A'$", (1.61,9.93),NE*lsf); dot((2.06,4.76),ds); label("$D'$", (2.2,4.96),NE*lsf); dot((2.39,2.41),ds); label("$B'$", (2.51,2.61),NE*lsf); dot((3.27,-3.8),ds); label("$C'$", (3.53,-3.91),NE*lsf); clip((-5.43,-5.18)--(-5.43,10.64)--(14.29,10.64)--(14.29,-5.18)--cycle);[/asy]
Note: Such four concurrent lines are called to form a harmonic range or a harmonic pencil, denoted as $O(ACBD)$ or sometimes as $O(AB; CD).$
Theorem 4.
Any two of the following statements implore the third statement to be true, too.
Statement 1: The pencil $O(AB; CD)$ is harmonic.
Statement 2: $OB$ bisects $\angle AOC$ internally.
Statement 3: $OB$ and $OD$ are perpendicular to each other.
Proof.
I will prove that $(1), (3)\implies (2),$ and the rest is left for the reader to prove.
Refer to the diagram. Draw $EF\parallel AO$ such that $E\in OC, F\in OD.$ Now,
$AO\parallel EF\implies \frac{AC}{CB}=\frac{AO}{EB};$ and $AO\parallel BF\implies \frac{AD}{BD}=\frac{AO}{BF}.$
Using $\frac{AC}{CB}=\frac{AD}{BD},$ we get $EB=BF,$ ie $B$ is the midpoint of $EF.$
Now, we already have $\angle OBF=\angle BOA=\frac{\pi}{2}\land EB=BF\implies \triangle OBE\cong\triangle OBF.$ Therefore $OB$ bisects $\angle EOF,$ but since this is perpendicular to $OA,$ therefore $OA$ is the external bisector of $\angle EOF.$
[asy]
import graph; size(300); real lsf = 0.5; pen dp = linewidth(0.5) + fontsize(10); defaultpen(dp); pen ds = black; pen xdxdff = rgb(0.49,0.49,1); pen wwqqff = rgb(0.4,0,1); pen zzzzff = rgb(0.6,0.6,1); pen ffttqq = rgb(1,0.2,0); pen zzwwff = rgb(0.6,0.4,1); pen ffqqww = rgb(1,0,0.4);
draw((-0.82,8.97)--(0.21,8.15)--(1.03,9.18)--(0,10)--cycle,wwqqff); draw((-12,0)--(-5,0),wwqqff); draw((0,10)--(-12,0),wwqqff); draw((0,10)--(-5,0),wwqqff); draw((-5,0)--(12.63,0),wwqqff); draw((0,10)--(12.63,0),wwqqff); draw((0,10)--(-7.92,0),wwqqff); pair parametricplot0_cus(real t){
return (2.79*cos(t)+0,2.79*sin(t)+10);
}
draw(graph(parametricplot0_cus,-2.2406491565943965,-2.0344439357957027)--(0,10)--cycle,ffttqq+linewidth(0.6pt)); pair parametricplot1_cus(real t){
return (2.79*cos(t)+0,2.79*sin(t)+10);
}
draw(graph(parametricplot1_cus,-2.44685437739309,-2.2406491565943965)--(0,10)--cycle,ffttqq); draw((-7.92,0)--(-13.59,-7.16),zzwwff); draw((3.59,7.16)--(-13.59,-7.16),ffqqww);
dot((-12,0),ds); label("$A$", (-12.25,-0.7),NE*lsf); dot((-5,0),ds); label("$B$", (-5.12,-0.75),NE*lsf); dot((0,10),ds); label("$O$", (-0.09,10.39),NE*lsf); dot((-7.92,0),ds); label("$C$", (-8.15,-0.75),NE*lsf); dot((12.63,0),ds); label("$D$", (12.44,-0.84),NE*lsf); dot((-13.59,-7.16),ds); label("$E$", (-13.09,-7.78),NE*lsf); dot((3.59,7.16),ds); label("$F$", (3.78,7.45),NE*lsf); clip((-15,-8.48)--(-15,15.28)--(14.63,15.28)--(14.63,-8.48)--cycle);[/asy]
Theorem 5.
If $(AB; CD)$ and $(A'B'; C'D')$ are harmonic, and if $AA', CC', BB'$ concur, then the line $DD'$ is also concurrent with these lines.
Proof.
Let $OD'$ meet $BC$ at some point $D'.$ We obtain that $O(A'B'C'D')$ is harmonic, implying $(AB, CD')$ is also harmonic. So we get
$\frac{AC}{CB}=\frac{AD'}{BD'}.$ But also $\frac{AC}{CB}=\frac{AD}{BD};$ so that
$\frac{AD'}{BD'}=\frac{AD}{BD}\implies \frac{AD'}{AD'-BD'}=\frac{AD}{AD-BD}\implies AD=AD',$ so that $D'$ and $D$ are coincident.
http://oi51.tinypic.com/27zyw7a.jpg
Theorem 6. (NEW)
Let $ABCD$ be a convex quadrilateral. If $K=AD\cap BC, M=AC\cap BD, P=AB\cap KM, Q=DC\cap KM,$ then we must have $K,M,P,Q$ to form a harmonic division.
Proof.
Let $R=BQ\cap AK.$ Then from Theorem 2 in $\triangle KDC$ with cevians $KQ,CA,BD$ and transversal $RQB,$ we observe that the division $K,R,D,A$ is harmonic.
So the range $B(K,R,D,A)$ is harmonic and using $KP$ as a transversal, we see that $(KMPQ)=-1.$ We are done.
[asy]
import graph; size(14cm); real lsf = 0.5; pen dp = linewidth(0.7) + fontsize(10); defaultpen(dp); pen ds = black; pen wwwwff = rgb(0.4,0.4,1); pen ffcccc = rgb(1,0.8,0.8);
draw((0,4)--(-2,0)--(4,0)--(2.02,3.66)--cycle); draw((0,4)--(-2,0),wwwwff); draw((-2,0)--(4,0),wwwwff); draw((4,0)--(2.02,3.66),wwwwff); draw((2.02,3.66)--(0,4),wwwwff); draw((-2,(+8+4*-2)/2)--(6.39,(+8+4*6.39)/2),wwwwff); draw((-2.92,(-14.64+3.66*-2.92)/-1.98)--(4,(-14.64+3.66*4)/-1.98),wwwwff); draw((-2,0)--(2.02,3.66),wwwwff); draw((4,0)--(0,4),wwwwff); draw((0.88,(+4.05-2.9*0.88)/0.26)--(6.39,(+4.05-2.9*6.39)/0.26),wwwwff); draw((-2.92,(-15.29+3.82*-2.92)/-2.95)--(4,(-15.29+3.82*4)/-2.95),wwwwff);
dot((0,4),ds); label("$D$", (-0.23,4.02),W*lsf); dot((-2,0),ds); label("$A$", (-2.17,-0.24),SW*lsf); dot((4,0),ds); label("$B$", (4.02,-0.3),NE*lsf); dot((2.02,3.66),ds); label("$C$", (2.07,3.74),NE*lsf); dot((0.88,5.76),ds); label("$K$", (0.99,5.72),NE*lsf); dot((1.14,2.86),ds); label("$M$", (0.87,2.76),W*lsf); dot((1.05,3.82),ds); label("$Q$", (1.11,3.91),NE*lsf); dot((1.4,0),ds); label("$P$", (1.44,-0.24),SE*lsf); dot((0.36,4.72),ds); label("$R$", (0.3,4.89),N*lsf); clip((-2.92,-1.31)--(-2.92,6.16)--(6.39,6.16)--(6.39,-1.31)--cycle);
[/asy]
$\Box$
Now we have already done some easy theorems which might come in handy here and there, so we need to keep a tab on them. Let's move onto some simple applications.
Note: From now on I might denote $(ACBD)$ as the cross-ratio and $(ACBD)=-1$ obviously denotes the case when $(AB; CD)$ is harmonic.
Note 2: I assume the reader to have some knowledge on Poles and Polars, Inversion, Homothety and Cross ratio. So hope I will not be too ambiguous while using these notations and notions. :lol:

$\boxed{E1}.$ If $AD,BE,CF$ are the altitudes of a triangle $\triangle ABC,$ and if $DE, EF$ meet $AB, BC$ at $F', D'$ respectively, then show that $FD$ and $F'D'$ intersect on a point lying on $AC.$
Solution
Let $FD$ meet $AC$ in $E',$ then we have to show that $D',E',F'$ are collinear.
Using Theorem 2, we have that $(BDCD')$ is harmonic, ie $\frac{BD}{DC}=\frac{BD'}{CD'}.$
Similarly we obtain three other relations, and multiplying them out we get
$\frac{BD'}{D'C}\cdot\frac{CE'}{E'A}\cdot\frac{AF'}{F'C}=\frac{BD}{DC}\cdot\frac{CE}{EA}\cdot\frac{AF}{FB}\stackrel{\text{Ceva}}{=}-1;$
So that using the converse of Menelaus theorem, $D'E'F'$ is a transversal to the triangle $\triangle ABC,$ and therefore a straight line.
Note: Even if I used directed segments in this proof, I think it is best to avoid using them.
[asy]   
import graph; size(300); real lsf = 0.5; pen dp = linewidth(0.5) + fontsize(10); defaultpen(dp); pen ds = black; pen wwqqff = rgb(0.4,0,1); pen zzttqq = rgb(0.6,0.2,0); pen ffwwww = rgb(1,0.4,0.4); pen qqzzzz = rgb(0,0.6,0.6);
draw((0,8)--(-2,0)--(4,0)--cycle); draw((0,8)--(-2,0),zzttqq); draw((-2,0)--(4,0),zzttqq); draw((4,0)--(0,8),zzttqq); draw((4,0)--(-8,0),wwqqff); draw((-8,0)--(7,-6),wwqqff); draw((0,8)--(7,-6),wwqqff); draw((2.8,2.4)--(-8,0),wwqqff); draw((0,8)--(-2.55,-2.18),wwqqff); draw((0,8)--(0,0),ffwwww); draw((-2,0)--(2.8,2.4),ffwwww); draw((4,0)--(-1.65,1.41),ffwwww); draw((-1.65,1.41)--(7,-6),qqzzzz); draw((2.8,2.4)--(-2.55,-2.18),wwqqff);
dot((-2,0),ds); label("$B$", (-2.56,-0.45),NE*lsf); dot((4,0),ds); label("$C$", (4.44,-0.36),NE*lsf); dot((0,8),ds); label("$A$", (-0.12,8.28),NE*lsf); dot((0,0),ds); label("$D$", (-0.12,-0.52),NE*lsf); dot((2.8,2.4),ds); label("$E$", (2.78,2.66),NE*lsf); dot((-1.65,1.41),ds); label("$F$", (-1.82,1.58),NW*lsf); dot((-2.55,-2.18),ds); label("$F'$", (-2.9,-2.89),NE*lsf); dot((-8,0),ds); label("$D'$", (-8.33,-0.45),S*lsf); dot((7,-6),ds); label("$E'$", (7.22,-6.01),NE*lsf); clip((-9.6,-6.75)--(-9.6,8.99)--(10.03,8.99)--(10.03,-6.75)--cycle);[/asy]

$\boxed{E2}.$ The tangent at a point $P$ of a circle cuts a diametre $AB$ at $T$; and $PN$ is the perpendicular to $AB,$ the line joining $B$ to the midpoint of $TP$ cuts $PN$ at $Q.$ Prove that $AQ\parallel TP.$
Solution
Let us assume $QA\cap TP=X.$ Note that we have, from the similarity of $\triangle PBN$ and $\triangle PBA;$ that
$\angle BPT=\angle BAP=\angle NPB.$ So using Theorem 4, we obtain that $(ABTN)$ is harmonic. So the pencil $Q(ABTN)$ is also harmonic, leading to (suing $TX$ as a transversal) $(XMTP)$ is harmonic.
But, this will be harmonic if and only if $\frac{XT}{MT}=\frac{XP}{PM}=\frac{XD}{TM}.$ But using $PM=TM,$ we get $XT=XP,$ but $T\neq P.$ Therefore $X=\infty,$ and $AQ\parallel TP.$
[asy]  
import graph; size(300); real lsf = 0.5; pen dp = linewidth(0.7) + fontsize(10); defaultpen(dp); pen ds = black; pen wwqqff = rgb(0.4,0,1); pen ffwwww = rgb(1,0.4,0.4); pen wwwwff = rgb(0.4,0.4,1); pen qqzzzz = rgb(0,0.6,0.6);
draw(circle((0,0),3),ffwwww); draw((-3,0)--(1.82,-3.68),wwwwff); draw((-3,0)--(4.93,0),wwwwff); draw((1.82,2.38)--(1.82,-3.68),qqzzzz); draw((1.82,2.38)--(4.93,0),wwwwff); draw((3.38,1.19)--(1.82,-3.68),wwwwff); draw((1.82,2.38)--(-3,0),wwwwff); draw((3,0)--(1.82,2.38),wwwwff);
dot((0,0),ds); label("$O$", (-0.06,-0.3),NE*lsf); dot((1.82,2.38),ds); label("$P$", (1.88,2.47),NE*lsf); dot((-3,0),ds); label("$A$", (-3.2,-0.25),W*lsf); dot((3,0),ds); label("$B$", (3.03,-0.25),NE*lsf); dot((4.93,0),ds); label("$T$", (4.84,-0.3),NE*lsf); dot((1.82,0),ds); label("$N$", (1.86,-0.28),NE*lsf); dot((3.38,1.19),ds); label("$M$", (3.49,1.18),NE*lsf); dot((1.82,-3.68),ds); label("$Q$", (1.91,-3.76),NE*lsf); clip((-3.91,-4.09)--(-3.91,3.75)--(5.87,3.75)--(5.87,-4.09)--cycle);
[/asy]

$\boxed{E4}$Let $ABC$ be a right angled triangle at $A$. $D$ is a point on $CB$. Let $M$ be the midpoint of $AD$. $CM$ intersects the perpendicular bisector of $AB$ at $E$. Prove that $BE\parallel DA$.
Solution
Let $K=AB\cap CE.$ We will show that if $E$ is a point on $CM$ extended such that $BE\parallel DA,$ Then we must have $L$ is the perpendicular from $E$ onto $AB\implies AE=EB.$
Now let $DA$ and $BE$ meet at $\infty,$ then; since $M$ is the midpoint of $AD$ it is forced that $(DMA\infty)$ is harmonic.
Now this also implies that $B(DMA\infty)$ is a harmonic range, so that using $CE$ as a transversal to this range it is implored that $C,M,K,E$ are harmonic.
Now since $\angle CAK = 90^{\circ}$ we must have $\angle MAK=\angle KAE.$ Therefore using $DA\parallel BE$ we have $\angle DAB=\angle LBE,$ and therefore $\triangle LAE\cong\triangle BEL.$
Hence $L$ is the midpoint of $AB.\Box$
[asy]
import graph; size(10cm);
draw((0,0)--(4,0));
draw((4,0)--(0,5));
draw((0,0)--(0,5));
draw((0,5)--(2,-2.51));
draw((0,0)--(2,2.5));
draw((0,0)--(2,-2.51));
draw((4,0)--(2,-2.51));
draw((2,0)--(2,-2.51));
label("$A$", (0,0), W);
label("$B$", (4,0), S);
label("$C$", (0,5), N);
label("$D$", (2,2.25), NE);
label("$E$", (2,-2.51), E);
label("$M$", (1,1.25), W);
label("$L$", (2,0), N);
[/asy]
$\boxed{E5}$In acute triangle $ABC,$ we have points $D$ and $E$ on sides $AC, AB$ respectively satisfying $\angle ADE=\angle ABC.$ Let the angle bisector of $\angle A$ meet $BC$ at $K.\ P$ and $L$ are projections of $K$ and $A$ to $DE,$ respectively, and $Q$ is the midpoint of $AL.$ If the incenter of $\triangle ABC$ lies on the circumcircle of $\triangle ADE,$ prove that $P,\ Q,$ and the incenter of $\triangle ADE$ are collinear.
Solution(motal)
Let $J\equiv AK\cap BD$, $R$ the incenter of $\triangle ABC$ and $I$ the incenter of $\triangle ADE$. $(AQL\infty)=-1$ so, considering $P(AQL\infty)$ with transversal $AK$, the problem is equivalent to proving that $(AIJK)=-1$. Easy angle chasing leads to $\triangle ERD$ isosceles. Furthermore, $\angle{ARD}=\angle{AED}=\gamma\Longrightarrow RKCD$ concyclic. With similar argument we obtain $BERK$ concyclic. $\angle{EID}+\angle{EKD}=180^{\circ}-\frac{\beta}{2}-\frac{\gamma}{2}+(\frac{\beta}{2}+\frac{\gamma}{2})=180^{\circ}$. Therefore we have that $IEKD$ is concyclic with diameter $KI$. In particular we have that both of these facts hold: $\angle{IDK}=90^{\circ}$ and $DI$ is bisector of $\angle{JDA}$. Hence considering pencil $D(AIJK)$ we can conclude that $(AIJK)=-1$.
We are done. $\Box$
Sorry I could not attach a figure, I will attach one as soon as possible.

$\boxed{E6}.$(Desargues) Let $\triangle ABC$ and $\triangle A'B'C'$ be perspective triangles perspective about a point $O.$ If $BC\cap B'C'=X, CA\cap C'A'=Y$ and $AB\cap A'B'=Z,$ then $X,Y,Z$ are collinear.
Solution
Let us denote $OA\cap BC=E, OA\cap B'C'=E'.$ Now we will use a little of cross ratio or projective geometry to prove this fact.
Firstly consider the concurrent points $B,E,C,Y$ and $B',E',C',Y.$ The corresponding lines $BB', EE', CC', YY$ are concurrent at $O.$ Therefore using a simple idea of cross-ratio we obtain that
$O(BY, EC)=O(B'Y, E'C).$ It is also obvious that $O(BY,EC)=A(BY, EC)=A(X,Y,Z,O)$ and $O(B'Y,E'C')=A'(X,Y,Z,O).$ So the points $X,Y,Z$ are collinear since $O$ lies on the line $AA'.$ (Why?)
We are done. :)
http://oi56.tinypic.com/zxlcvp.jpg

$\boxed{E7}.$Given that $AB$ is a diameter of a circle and the lines $CD,CB$ are tangents to the circle, prove that $DE = EF,$ where $F$ is the foot of perpendicular from $D$ onto $AB$ and $E=DF\cap CA.$
Solution
Let $U=CD\cap BA$ and $S=BD\cap CA.$
Since $DF\perp AB$ and $BD\perp DA,$ we see that $\angle ADU=\angle ABD=\angle FDA,$ therefore $DA$ bisects $\angle FDU$ and also $AD\perp BD.$ So the range $D(B,F,A,U)$ is harmonic. Using $CA$ as a transversal this leads to the fact that $S,E,A,C$ are harmonic. So $B(S,E,A,C)$ is harmonic, and again, using $DF$ as a transversal to this range we have that $(FED\infty)=-1,$ ie $E$ is the midpoint of $DF.$
These types of midpoint considerations can prove a lot of geometric problems easily, and of course this one is obvious from considering symmedians. Apart from that, harmonic divisions stands out to be a hugely useful and rarely used (in my country, at least) method.

$\boxed{E8}.$(Proposed in Olympiad Marathon by Fersolve)
Let $O$ be the intersection of the diagonals of convex quadrilateral $ABCD.$ The circumcircles
of $\triangle OAD$ and $\triangle OBC$ meet at $O$ and $M.$ Line $OM$ meets the circumcircles of $\triangle OAB$ and $\triangle OCD$ at $S$ and $T$ respectively. Prove that $M$ is the midpoint of $ST.$
(NEW)
[asy]import graph; size(14cm); real lsf = 0.5; pen dp = linewidth(0.7) + fontsize(10); defaultpen(dp); pen ds = black; pen qqttff = rgb(0,0.2,1); pen ffcccc = rgb(1,0.8,0.8); pen fftttt = rgb(1,0.2,0.2);
draw((0,5)--(-2,0)--(6,0)--(2.31,6.49)--cycle); draw((0,5)--(-2,0),qqttff); draw((-2,0)--(6,0),qqttff); draw((6,0)--(2.31,6.49),qqttff); draw((2.31,6.49)--(0,5),qqttff); draw((-2,0)--(2.31,6.49),qqttff); draw((0,5)--(6,0),qqttff); draw(circle((-1.27,2.61),2.71),fftttt); draw(circle((4.51,3.45),3.76),fftttt); draw((-5.22,(-3.97+2.69*-5.22)/-0.39)--(10.02,(-3.97+2.69*10.02)/-0.39),qqttff); draw(circle((1.24,5.62),1.38),fftttt); draw(circle((2,0.44),4.02),fftttt);
dot((0,5),ds); label("$D$", (-0.43,5.18),NW*lsf); dot((-2,0),ds); label("$A$", (-2.34,-0.36),W*lsf); dot((6,0),ds); label("$B$", (6.14,-0.36),NE*lsf); dot((2.31,6.49),ds); label("$C$", (2.4,6.64),NE*lsf); dot((0.85,4.29),ds); label("$O$", (1.18,4.03),NE*lsf); dot((1.24,1.6),ds); label("$M$", (1.37,1.61),NE*lsf); dot((0.49,6.78),ds); label("$T$", (0.56,7.02),NE*lsf); dot((1.99,-3.59),ds); label("$S$", (2.09,-3.45),NE*lsf); clip((-5.22,-4.1)--(-5.22,8.12)--(10.02,8.12)--(10.02,-4.1)--cycle);[/asy]
Solution
Let us invert everything about $O$ to map points to their primes. By Theorem 6 on the four collinear points $M',T',S',O;$ we have $\dfrac{M'T'}{T'O}=\dfrac{M'S'}{S'O}.$ Also we have the following relations:
\[M'T'=\dfrac{MT\times r^2}{OM\cdot OT}, T'O=\dfrac{r^2}{OT}, M'S'=\dfrac{MS\times r^2}{OM\cdot OS}, S'O=\dfrac{r^2}{OS};\]And therefore we obtain $MT=MS$ and so, $M$ is the midpoint of $TS$ as required.$\Box$

I will keep updating this post from time to time because as time goes on, the number of problems you come across also goes on.
I will add some exercises about a month later (after my secondary exams end), so, till then; Rejoice!!
References
[1] Cosmin Pohoata; Harmonic Divisions; 2007 Mathematical Reflections
[2] C. Stanley Ogilvy; (1990) Excursions in Geometry, Dover.
[3] Application 1
[4] Application 2
:)
Last Updated On: 7:00 pm UCT; 03 March, 2011.
This post has been edited 6 times. Last edited by levans, Jan 15, 2016, 3:57 AM

Comment

7 Comments

The post below has been deleted. Click to close.
This post has been deleted. Click here to see post.
Oh NO!, I was thinking of starting a cross ratio blog. But this is too much better than mine with all those asymptote images.
In theorem $2$, should $AX, BY, CZ$ be concurrent? Because the diagram mentions that while the proof doesn't,.
And cross ratios are far reaching than harmonic division.

K, next time you start a blog, don't put Isogonal conjugates because I am going to do precisely that.

And you should give some exercises.
This post has been edited 1 time. Last edited by Goutham, Feb 9, 2011, 1:21 AM

by Goutham, Feb 9, 2011, 1:20 AM

The post below has been deleted. Click to close.
This post has been deleted. Click here to see post.
They are cevians, and therefore concurrent. :lol:
And, yeah, cross ratios are really powerful, but I am not very used to it - I am learning things step-by-step.
So I think you should go on and post your ideas on harmonic divisions in your blog, I wanna see it.
Also it would help very much if you include cross ratios and Isogonal conjugates and Apollonian Properties, because I am not very comfortable with these ideas.
Thanks a lot for commenting. :)

by Potla, Feb 9, 2011, 4:46 AM

The post below has been deleted. Click to close.
This post has been deleted. Click here to see post.
I don't think cevians mean they have to be concurrent.

And my blog will take time, I am busy gaming and watching anime!

by Goutham, Feb 9, 2011, 8:31 AM

The post below has been deleted. Click to close.
This post has been deleted. Click here to see post.
Wow, very nice! :) :cool:

by Amir Hossein, Feb 9, 2011, 12:28 PM

The post below has been deleted. Click to close.
This post has been deleted. Click here to see post.
Thanks Potla :lol:

There is also another good reference for harmonic division.

See it $\rightarrow$ http://www.artofproblemsolving.com/Forum/viewtopic.php?f=50&t=161310

by Polymath1729, Feb 9, 2011, 5:15 PM

The post below has been deleted. Click to close.
This post has been deleted. Click here to see post.
Actually E1 is a corolary from Desargue's theorem and it is not needed that the cevians are the altitudes in the triangle :)

by oups_GFX, May 7, 2011, 9:03 PM

The post below has been deleted. Click to close.
This post has been deleted. Click here to see post.
Potla, for the last problem, I can see a more simpler proof using inversion.
Note that, $ (M'T',OS')=-1 $. We know that inversion does not change the cross-ratio.
So $(MT,\infty S)=-1 $ since under inversion wrt $ O $ the point at infinity comes to $ O $.
Clearly this implies, $ S $ is mid-point of $ MT $.

by RSM, Jun 22, 2011, 12:51 PM

Compiled problems

avatar

Potla
Shouts
Submit
  • first shouts in 2025!

    by NicoN9, Apr 25, 2025, 12:56 PM

  • dang 2 year bump (can the person who next visits PM me? I want to see when the next person visits)

    by roribaki, Jan 6, 2024, 9:18 PM

  • wholsome

    by samrocksnature, Aug 24, 2021, 4:16 PM

  • wholesome

    by centslordm, Jul 2, 2021, 2:08 PM

  • :omighty: cool blog

    by lneis1, Feb 20, 2021, 6:11 AM

  • Hello.Very nice blog!

    by Functional_equation, Aug 29, 2020, 2:20 PM

  • Great blog!

    by freeman66, Jun 3, 2020, 1:25 AM

  • Hello. Nice blog!

    by mathboy282, Apr 10, 2020, 1:39 AM

  • Masterpiece

    by Kamran011, Mar 18, 2020, 6:35 PM

  • Hello! Why am I looking through random blogs?

    by bingo2016, Nov 21, 2019, 2:40 AM

  • I am late to find this wonderful gem

    by RAMUGAUSS, Nov 13, 2019, 7:32 AM

  • I WILL NECROPOST
    I WILL NECROPOST
    I WILL NECROPOST
    I WILL NECROPOST
    I WILL NECROPOST
    I WILL NECROPOST
    I WILL NECROPOST
    I WILL NECROPOST
    I WILL NECROPOST
    I WILL NECROPOST
    I WILL NECROPOST
    I WILL NECROPOST
    I WILL NECROPOST
    I WILL NECROPOST
    I WILL NECROPOST

    by Davis1234567890, Sep 15, 2019, 2:21 AM

  • This is a nice blog and it is very useful!

    by arandomperson123, Jun 7, 2016, 2:32 AM

  • bump,,,,

    by ythomashu, Jan 15, 2016, 12:51 AM

  • Incredible

    by DanielL2000, Jul 24, 2014, 9:34 PM

59 shouts
Contributors
Tags
About Owner
  • Posts: 1886
  • Joined: Nov 28, 2008
Blog Stats
  • Blog created: Feb 13, 2009
  • Total entries: 25
  • Total visits: 105103
  • Total comments: 83
Search Blog
a